Question and Answers Forum

All Questions      Topic List

Relation and Functions Questions

Previous in All Question      Next in All Question      

Previous in Relation and Functions      Next in Relation and Functions      

Question Number 35688 by prof Abdo imad last updated on 22/May/18

let A_n  =Σ_(k=1) ^n    (1/(k+n))ln(1+(k/n))  calculate lim_(n→+∞) A_n

$${let}\:{A}_{{n}} \:=\sum_{{k}=\mathrm{1}} ^{{n}} \:\:\:\frac{\mathrm{1}}{{k}+{n}}{ln}\left(\mathrm{1}+\frac{{k}}{{n}}\right) \\ $$$${calculate}\:{lim}_{{n}\rightarrow+\infty} {A}_{{n}} \\ $$

Commented by prof Abdo imad last updated on 22/May/18

we have A_n   = Σ_(k=1) ^n   (1/(n( (k/n) +1)))ln(1+(k/n))  = (1/n) Σ_(k=1) ^n     ((ln(1+(k/n)))/(1+(k/n))) so A_n   is a Rieman sum  lim_(n→+∞)  A_n  = ∫_0 ^1   ((ln(1+x))/(1+x))dx =I   by parts  I  =[ ln^2 (1+x)]_0 ^1  −∫_0 ^1    ((ln(1+x))/(1+x))dx  = {ln(2)}^2  −I ⇒ 2I = {ln(2)}^2  ⇒  I =(({ln(2)}^2 )/2) =lim_(n→+∞)  A_n   ⇒

$${we}\:{have}\:{A}_{{n}} \:\:=\:\sum_{{k}=\mathrm{1}} ^{{n}} \:\:\frac{\mathrm{1}}{{n}\left(\:\frac{{k}}{{n}}\:+\mathrm{1}\right)}{ln}\left(\mathrm{1}+\frac{{k}}{{n}}\right) \\ $$$$=\:\frac{\mathrm{1}}{{n}}\:\sum_{{k}=\mathrm{1}} ^{{n}} \:\:\:\:\frac{{ln}\left(\mathrm{1}+\frac{{k}}{{n}}\right)}{\mathrm{1}+\frac{{k}}{{n}}}\:{so}\:{A}_{{n}} \:\:{is}\:{a}\:{Rieman}\:{sum} \\ $$$${lim}_{{n}\rightarrow+\infty} \:{A}_{{n}} \:=\:\int_{\mathrm{0}} ^{\mathrm{1}} \:\:\frac{{ln}\left(\mathrm{1}+{x}\right)}{\mathrm{1}+{x}}{dx}\:={I} \\ $$$$\:{by}\:{parts}\:\:{I}\:\:=\left[\:{ln}^{\mathrm{2}} \left(\mathrm{1}+{x}\right)\right]_{\mathrm{0}} ^{\mathrm{1}} \:−\int_{\mathrm{0}} ^{\mathrm{1}} \:\:\:\frac{{ln}\left(\mathrm{1}+{x}\right)}{\mathrm{1}+{x}}{dx} \\ $$$$=\:\left\{{ln}\left(\mathrm{2}\right)\right\}^{\mathrm{2}} \:−{I}\:\Rightarrow\:\mathrm{2}{I}\:=\:\left\{{ln}\left(\mathrm{2}\right)\right\}^{\mathrm{2}} \:\Rightarrow \\ $$$${I}\:=\frac{\left\{{ln}\left(\mathrm{2}\right)\right\}^{\mathrm{2}} }{\mathrm{2}}\:={lim}_{{n}\rightarrow+\infty} \:{A}_{{n}} \\ $$$$\Rightarrow\: \\ $$

Answered by tanmay.chaudhury50@gmail.com last updated on 22/May/18

=((lim)/(n→∞))×(1/n)×(1/(1+((k/n))))×ln(1+(k/n))  =∫_0 ^1 ((ln(1+x))/(1+x))×dx  t=ln(1+x)   dt=(dx/(1+x))   ∫_0 ^(ln2) tdt  =∣(t^2 /2)∣_0 ^(ln2)   =(((ln2)^2 )/2)

$$=\frac{{lim}}{{n}\rightarrow\infty}×\frac{\mathrm{1}}{{n}}×\frac{\mathrm{1}}{\mathrm{1}+\left(\frac{{k}}{{n}}\right)}×{ln}\left(\mathrm{1}+\frac{{k}}{{n}}\right) \\ $$$$=\int_{\mathrm{0}} ^{\mathrm{1}} \frac{{ln}\left(\mathrm{1}+{x}\right)}{\mathrm{1}+{x}}×{dx} \\ $$$${t}={ln}\left(\mathrm{1}+{x}\right)\:\:\:{dt}=\frac{{dx}}{\mathrm{1}+{x}}\: \\ $$$$\int_{\mathrm{0}} ^{{ln}\mathrm{2}} {tdt} \\ $$$$=\mid\frac{{t}^{\mathrm{2}} }{\mathrm{2}}\mid_{\mathrm{0}} ^{{ln}\mathrm{2}} \\ $$$$=\frac{\left({ln}\mathrm{2}\right)^{\mathrm{2}} }{\mathrm{2}} \\ $$

Commented by prof Abdo imad last updated on 22/May/18

sir Tanmays your answer is correct thanks...

$${sir}\:{Tanmays}\:{your}\:{answer}\:{is}\:{correct}\:{thanks}... \\ $$

Terms of Service

Privacy Policy

Contact: info@tinkutara.com